Sum of two continuous Uniform $(0$,$1)$ random variables without convolutionA question about independence of bivariate random variablesRatio of Gamma random variablesFinding the joint probability density function of two random variablesProbability Theory - Transformation of independent continuous random variablesTransformation of continuous, independent random variablesJoint probability distribution of functions of random variablesIndependence of some derived random variablesFinding the joint distribution of two random variables (Normal distribution)Show that $Y_1$ and $Y_2$ are independent $N(0, 1)$-distributed random variables.Sum of two multinomial random variables

Is it true that good novels will automatically sell themselves on Amazon (and so on) and there is no need for one to waste time promoting?

Should I use acronyms in dialogues before telling the readers what it stands for in fiction?

Do I need to consider instance restrictions when showing a language is in P?

Pronounciation of the combination "st" in spanish accents

How to terminate ping <dest> &

Optimising a list searching algorithm

Practical application of matrices and determinants

Can a wizard cast a spell during their first turn of combat if they initiated combat by releasing a readied spell?

Help rendering a complicated sum/product formula

PTIJ What is the inyan of the Konami code in Uncle Moishy's song?

In the 1924 version of The Thief of Bagdad, no character is named, right?

Can a medieval gyroplane be built?

Knife as defense against stray dogs

The average age of first marriage in Russia

Maths symbols and unicode-math input inside siunitx commands

How do hiring committees for research positions view getting "scooped"?

What does Jesus mean regarding "Raca," and "you fool?" - is he contrasting them?

My friend is being a hypocrite

How are passwords stolen from companies if they only store hashes?

What is the term when voters “dishonestly” choose something that they do not want to choose?

World War I as a war of liberals against authoritarians?

Variable completely messes up echoed string

Existence of a celestial body big enough for early civilization to be thought of as a second moon

Worshiping one God at a time?



Sum of two continuous Uniform $(0$,$1)$ random variables without convolution


A question about independence of bivariate random variablesRatio of Gamma random variablesFinding the joint probability density function of two random variablesProbability Theory - Transformation of independent continuous random variablesTransformation of continuous, independent random variablesJoint probability distribution of functions of random variablesIndependence of some derived random variablesFinding the joint distribution of two random variables (Normal distribution)Show that $Y_1$ and $Y_2$ are independent $N(0, 1)$-distributed random variables.Sum of two multinomial random variables













2












$begingroup$


We can transform one continuous multivariate distribution to another based on two chosen transformation functions, their inverses and derivatives. These course notes go through the process in detail, but to cut to the chase:



  • Start with two random variables $X_1$ and $X_2$.

  • Assume the associated bivariate probability density function is $f(x_1, x_2)$.

  • Choose two transformation functions $y_1(x_1, x_2)$ and $y_2(x_1, x_2)$.

  • Let the derived random variables be $Y_1 = y_1(X_1, X_2)$ and $Y_2 = y_2(X_1, X_2)$.

  • Assume the associated bivariate probability density function is $g(y_1, y_2$).

  • Assume the inverses of the two transformation functions are $x_1(y_1, y_2)$ and $x_2(y_1, y_2)$.

  • The relationship between $f(x_1, x_2)$ and $g(y_1, y_2)$ is:
    $$
    g(y_1, y_2) = f(x_1(y_1, y_2), x_2(y_1, y_2) cdot |J|
    $$


Where J is the determinant of the Jacobian matrix of $[x_1, x_2]$ with respect to $[y_1, y_2]$:



$$
J = frac∂(x_1, x_2)∂(y_1, y_2)
$$



  • As a special case, if $f(x1, x2)$ corresponds to a uniform
    distribution, the relationship is:

$$
g(y_1, y_2) = |J|
$$



Suppose that $X_1$ and $X_2$ are continuous i.i.d. random variables distributed Uniform(0,1).



Let $Y_1 = X_1 + X_2$ and $Y_2 = X_1 - X_2$



Note that $Y_2$ can be anything since it's only $Y_1$ we care about.



As far as I can tell, this setup satisfies the requirements of the process outlined above. The transformation functions have inverses, they determine a bijective mapping between the domain of $f$ and the domain of $g$.



So it should be possible to derive first the joint distribution $g$ of $Y_1$ and $Y_2$ and then the marginal distribution of $Y_1$, which we know is triangular.



But when I go through the process that's not what I get. In fact, my joint distribution (which is just $|J|$) ends up being a constant $1/2$, which doesn't sum to $1$ over its domain and therefore is not even a valid density function.



I know that normally we would derive the sum of two Uniform random variables using convolution, but I'd like to know if it's possible to use the above process (or if not, I'd like to know why).










share|cite|improve this question











$endgroup$







  • 2




    $begingroup$
    I'd say: Yes it's possible, and it should be absolutely equivalent to a convolution integral, in your case.
    $endgroup$
    – Matteo
    Mar 12 at 18:44











  • $begingroup$
    That's good news! I would love to see a worked example or at least a hint on where I'm going wrong because my calculation of $g$ is obviously not correct.
    $endgroup$
    – chris838
    Mar 13 at 12:42






  • 1




    $begingroup$
    As soon as I can I will write down an example. For now what I'm wondering is: are you sure that in your solution you didn't just miss to correct limits of integration? Because this is the main contribution to the integral in this case.
    $endgroup$
    – Matteo
    Mar 13 at 12:53















2












$begingroup$


We can transform one continuous multivariate distribution to another based on two chosen transformation functions, their inverses and derivatives. These course notes go through the process in detail, but to cut to the chase:



  • Start with two random variables $X_1$ and $X_2$.

  • Assume the associated bivariate probability density function is $f(x_1, x_2)$.

  • Choose two transformation functions $y_1(x_1, x_2)$ and $y_2(x_1, x_2)$.

  • Let the derived random variables be $Y_1 = y_1(X_1, X_2)$ and $Y_2 = y_2(X_1, X_2)$.

  • Assume the associated bivariate probability density function is $g(y_1, y_2$).

  • Assume the inverses of the two transformation functions are $x_1(y_1, y_2)$ and $x_2(y_1, y_2)$.

  • The relationship between $f(x_1, x_2)$ and $g(y_1, y_2)$ is:
    $$
    g(y_1, y_2) = f(x_1(y_1, y_2), x_2(y_1, y_2) cdot |J|
    $$


Where J is the determinant of the Jacobian matrix of $[x_1, x_2]$ with respect to $[y_1, y_2]$:



$$
J = frac∂(x_1, x_2)∂(y_1, y_2)
$$



  • As a special case, if $f(x1, x2)$ corresponds to a uniform
    distribution, the relationship is:

$$
g(y_1, y_2) = |J|
$$



Suppose that $X_1$ and $X_2$ are continuous i.i.d. random variables distributed Uniform(0,1).



Let $Y_1 = X_1 + X_2$ and $Y_2 = X_1 - X_2$



Note that $Y_2$ can be anything since it's only $Y_1$ we care about.



As far as I can tell, this setup satisfies the requirements of the process outlined above. The transformation functions have inverses, they determine a bijective mapping between the domain of $f$ and the domain of $g$.



So it should be possible to derive first the joint distribution $g$ of $Y_1$ and $Y_2$ and then the marginal distribution of $Y_1$, which we know is triangular.



But when I go through the process that's not what I get. In fact, my joint distribution (which is just $|J|$) ends up being a constant $1/2$, which doesn't sum to $1$ over its domain and therefore is not even a valid density function.



I know that normally we would derive the sum of two Uniform random variables using convolution, but I'd like to know if it's possible to use the above process (or if not, I'd like to know why).










share|cite|improve this question











$endgroup$







  • 2




    $begingroup$
    I'd say: Yes it's possible, and it should be absolutely equivalent to a convolution integral, in your case.
    $endgroup$
    – Matteo
    Mar 12 at 18:44











  • $begingroup$
    That's good news! I would love to see a worked example or at least a hint on where I'm going wrong because my calculation of $g$ is obviously not correct.
    $endgroup$
    – chris838
    Mar 13 at 12:42






  • 1




    $begingroup$
    As soon as I can I will write down an example. For now what I'm wondering is: are you sure that in your solution you didn't just miss to correct limits of integration? Because this is the main contribution to the integral in this case.
    $endgroup$
    – Matteo
    Mar 13 at 12:53













2












2








2





$begingroup$


We can transform one continuous multivariate distribution to another based on two chosen transformation functions, their inverses and derivatives. These course notes go through the process in detail, but to cut to the chase:



  • Start with two random variables $X_1$ and $X_2$.

  • Assume the associated bivariate probability density function is $f(x_1, x_2)$.

  • Choose two transformation functions $y_1(x_1, x_2)$ and $y_2(x_1, x_2)$.

  • Let the derived random variables be $Y_1 = y_1(X_1, X_2)$ and $Y_2 = y_2(X_1, X_2)$.

  • Assume the associated bivariate probability density function is $g(y_1, y_2$).

  • Assume the inverses of the two transformation functions are $x_1(y_1, y_2)$ and $x_2(y_1, y_2)$.

  • The relationship between $f(x_1, x_2)$ and $g(y_1, y_2)$ is:
    $$
    g(y_1, y_2) = f(x_1(y_1, y_2), x_2(y_1, y_2) cdot |J|
    $$


Where J is the determinant of the Jacobian matrix of $[x_1, x_2]$ with respect to $[y_1, y_2]$:



$$
J = frac∂(x_1, x_2)∂(y_1, y_2)
$$



  • As a special case, if $f(x1, x2)$ corresponds to a uniform
    distribution, the relationship is:

$$
g(y_1, y_2) = |J|
$$



Suppose that $X_1$ and $X_2$ are continuous i.i.d. random variables distributed Uniform(0,1).



Let $Y_1 = X_1 + X_2$ and $Y_2 = X_1 - X_2$



Note that $Y_2$ can be anything since it's only $Y_1$ we care about.



As far as I can tell, this setup satisfies the requirements of the process outlined above. The transformation functions have inverses, they determine a bijective mapping between the domain of $f$ and the domain of $g$.



So it should be possible to derive first the joint distribution $g$ of $Y_1$ and $Y_2$ and then the marginal distribution of $Y_1$, which we know is triangular.



But when I go through the process that's not what I get. In fact, my joint distribution (which is just $|J|$) ends up being a constant $1/2$, which doesn't sum to $1$ over its domain and therefore is not even a valid density function.



I know that normally we would derive the sum of two Uniform random variables using convolution, but I'd like to know if it's possible to use the above process (or if not, I'd like to know why).










share|cite|improve this question











$endgroup$




We can transform one continuous multivariate distribution to another based on two chosen transformation functions, their inverses and derivatives. These course notes go through the process in detail, but to cut to the chase:



  • Start with two random variables $X_1$ and $X_2$.

  • Assume the associated bivariate probability density function is $f(x_1, x_2)$.

  • Choose two transformation functions $y_1(x_1, x_2)$ and $y_2(x_1, x_2)$.

  • Let the derived random variables be $Y_1 = y_1(X_1, X_2)$ and $Y_2 = y_2(X_1, X_2)$.

  • Assume the associated bivariate probability density function is $g(y_1, y_2$).

  • Assume the inverses of the two transformation functions are $x_1(y_1, y_2)$ and $x_2(y_1, y_2)$.

  • The relationship between $f(x_1, x_2)$ and $g(y_1, y_2)$ is:
    $$
    g(y_1, y_2) = f(x_1(y_1, y_2), x_2(y_1, y_2) cdot |J|
    $$


Where J is the determinant of the Jacobian matrix of $[x_1, x_2]$ with respect to $[y_1, y_2]$:



$$
J = frac∂(x_1, x_2)∂(y_1, y_2)
$$



  • As a special case, if $f(x1, x2)$ corresponds to a uniform
    distribution, the relationship is:

$$
g(y_1, y_2) = |J|
$$



Suppose that $X_1$ and $X_2$ are continuous i.i.d. random variables distributed Uniform(0,1).



Let $Y_1 = X_1 + X_2$ and $Y_2 = X_1 - X_2$



Note that $Y_2$ can be anything since it's only $Y_1$ we care about.



As far as I can tell, this setup satisfies the requirements of the process outlined above. The transformation functions have inverses, they determine a bijective mapping between the domain of $f$ and the domain of $g$.



So it should be possible to derive first the joint distribution $g$ of $Y_1$ and $Y_2$ and then the marginal distribution of $Y_1$, which we know is triangular.



But when I go through the process that's not what I get. In fact, my joint distribution (which is just $|J|$) ends up being a constant $1/2$, which doesn't sum to $1$ over its domain and therefore is not even a valid density function.



I know that normally we would derive the sum of two Uniform random variables using convolution, but I'd like to know if it's possible to use the above process (or if not, I'd like to know why).







probability probability-distributions density-function






share|cite|improve this question















share|cite|improve this question













share|cite|improve this question




share|cite|improve this question








edited Mar 14 at 12:39







chris838

















asked Mar 12 at 18:39









chris838chris838

1134




1134







  • 2




    $begingroup$
    I'd say: Yes it's possible, and it should be absolutely equivalent to a convolution integral, in your case.
    $endgroup$
    – Matteo
    Mar 12 at 18:44











  • $begingroup$
    That's good news! I would love to see a worked example or at least a hint on where I'm going wrong because my calculation of $g$ is obviously not correct.
    $endgroup$
    – chris838
    Mar 13 at 12:42






  • 1




    $begingroup$
    As soon as I can I will write down an example. For now what I'm wondering is: are you sure that in your solution you didn't just miss to correct limits of integration? Because this is the main contribution to the integral in this case.
    $endgroup$
    – Matteo
    Mar 13 at 12:53












  • 2




    $begingroup$
    I'd say: Yes it's possible, and it should be absolutely equivalent to a convolution integral, in your case.
    $endgroup$
    – Matteo
    Mar 12 at 18:44











  • $begingroup$
    That's good news! I would love to see a worked example or at least a hint on where I'm going wrong because my calculation of $g$ is obviously not correct.
    $endgroup$
    – chris838
    Mar 13 at 12:42






  • 1




    $begingroup$
    As soon as I can I will write down an example. For now what I'm wondering is: are you sure that in your solution you didn't just miss to correct limits of integration? Because this is the main contribution to the integral in this case.
    $endgroup$
    – Matteo
    Mar 13 at 12:53







2




2




$begingroup$
I'd say: Yes it's possible, and it should be absolutely equivalent to a convolution integral, in your case.
$endgroup$
– Matteo
Mar 12 at 18:44





$begingroup$
I'd say: Yes it's possible, and it should be absolutely equivalent to a convolution integral, in your case.
$endgroup$
– Matteo
Mar 12 at 18:44













$begingroup$
That's good news! I would love to see a worked example or at least a hint on where I'm going wrong because my calculation of $g$ is obviously not correct.
$endgroup$
– chris838
Mar 13 at 12:42




$begingroup$
That's good news! I would love to see a worked example or at least a hint on where I'm going wrong because my calculation of $g$ is obviously not correct.
$endgroup$
– chris838
Mar 13 at 12:42




1




1




$begingroup$
As soon as I can I will write down an example. For now what I'm wondering is: are you sure that in your solution you didn't just miss to correct limits of integration? Because this is the main contribution to the integral in this case.
$endgroup$
– Matteo
Mar 13 at 12:53




$begingroup$
As soon as I can I will write down an example. For now what I'm wondering is: are you sure that in your solution you didn't just miss to correct limits of integration? Because this is the main contribution to the integral in this case.
$endgroup$
– Matteo
Mar 13 at 12:53










2 Answers
2






active

oldest

votes


















1












$begingroup$

Consider two independent random variables $X_1$ and $X_2$ with uniform distribution over the interval $[0,1]$. Thus we have
$$
f_X_i(x) =
begincases
1 & mboxif x in [0,1]\
0 & mboxotherwise
endcases
$$

for $i=1,2$.
Consider also, e.g., the transformation
begineqnarray
Y_1 &=& X_1+X_2;\
Y_2 &=& X_2;
endeqnarray

Then, as stated by your notes, the joint distribution of $Y_1$ and $Y_2$ is given by
$$
f_Y_1,Y_2(u,v) =
begincases
f_X_1,X_2(u-v,v) =f_X_1(u-v)cdot f_X_2(v) & mboxif (u,v)in [0,1]times[0,1]\
0 & mboxotherwise,
endcases
$$

where we used independence of $X_1$ and $X_2$.



Because you're interested only in the marginal distribution of $Y_1$ we get
$$
f_Y_1(u) = int_0^1 f_X_1(u-v)f_X_2(v) dv,
$$

that is exactly the convolution you were expecting.




EDIT As you requested, I little bit more details. If you use the transformations
begineqnarray
Y_1 &=& g_1(X_1,X_2)\
Y_2 &=& g_2(X_1,X_2)
endeqnarray

you first need to invert them and get
begineqnarray
X_1 &=& h_1(Y_1,Y_2)\
X_2 &=& h_2(Y_1,Y_2).
endeqnarray

Only now, I would recommend, can you



  1. Calculate, from $h_1(u,v)$ and $h_2(u,v)$, $$J = left|beginarraycc fracpartial h_1partial u & fracpartial h_1partial v \ fracpartial h_2partial u & fracpartial h_2partial vendarray right|$$

  2. Determine the joint distribution of $(Y_2,Y_1)$, that, limited to the correct domain, will be given by $$f_Y_1,Y_2(u,v) = |J|cdot f_X_1,X_2(h_1(u,v),h_2(u,v)).$$


In my example
begineqnarray
X_1 &=& Y_1-Y_2\
X_2 &=& Y_2
endeqnarray

are the inverse transformations, so that
begineqnarray
h_1(u,v) &=& u-v\
h_2(u,v) &=& v;
endeqnarray






share|cite|improve this answer











$endgroup$












  • $begingroup$
    Thanks! This makes sense and I think I see what I did wrong. I think I need a variable substitution in limits of my integral to get to the form you have here.
    $endgroup$
    – chris838
    Mar 14 at 12:47






  • 1




    $begingroup$
    @chris838 you're welcome! And: yes, but as you see it would be a little "convoluted" (...) to have a more complicated $Y_2$, since you have to marginalize it "out".
    $endgroup$
    – Matteo
    Mar 14 at 12:49











  • $begingroup$
    Yeh the $Y_2$ makes this much simpler in fact. I'm still stuck on how you get $f_X_1,X_2(u−v, v)$. I end up with $f_X_1,X_2((1/2)(u + v))$ - is that on the right track?
    $endgroup$
    – chris838
    Mar 14 at 13:38






  • 1




    $begingroup$
    @chris838 Ok, I added some more details at the bottom. Are my steps clearer now?
    $endgroup$
    – Matteo
    Mar 14 at 18:54






  • 1




    $begingroup$
    Thanks this is amazing!
    $endgroup$
    – chris838
    Mar 14 at 19:06


















0












$begingroup$

Yes, you can of course use that approach to get the joint density function, but I don't think your way would be easier to calculate in general.



Regarding your question on $g(y1, y2)$ not being a valid density function. You actually made a mistake here. $g(y_1, y_2)/f(x_1, x_2) = 1/|J|$ rather than $J$.



Just take an example of a single-variable density function ($C$ is the cumulative distribution function)
$$
p(x) = fracdCdx$
$$

$$
p(y) = fracdCdy = fracdCdx cdot fracdxdy = fracp(x)y'(x)
$$






share|cite|improve this answer









$endgroup$












  • $begingroup$
    Hmm, but the notes I refer to explicitly state $g(y_1,y_2) = |J|$ ??
    $endgroup$
    – chris838
    Mar 13 at 12:38











  • $begingroup$
    Possibly you're defining $J$ as $∂(y_1,y_2)/∂(x_1,x_2)$ whereas here I'm defining it as $∂(x_1,x_2)/∂(y_1,y_2)$
    $endgroup$
    – chris838
    Mar 13 at 12:48










  • $begingroup$
    Yes, you are right about the difference in our definition. However, in that case your calculation is wrong $partial(y_1,y_2)/partial(x_1, x_2)=2$ and $partial(x_1, x_2)/partial(y_1, y_2) = 1/2$.
    $endgroup$
    – MoonKnight
    Mar 13 at 16:44










  • $begingroup$
    Apologies - I was playing around with different transform functions. You're right and I've updated the question.
    $endgroup$
    – chris838
    Mar 14 at 12:40






  • 1




    $begingroup$
    your new integral area in $(Y_1, Y_2)$ space is a diamond with 4 cornors (0,0), (1,1), (-1,1), (2, 0), which area is $2$. so it does sum up to $1$.
    $endgroup$
    – MoonKnight
    Mar 14 at 16:47










Your Answer





StackExchange.ifUsing("editor", function ()
return StackExchange.using("mathjaxEditing", function ()
StackExchange.MarkdownEditor.creationCallbacks.add(function (editor, postfix)
StackExchange.mathjaxEditing.prepareWmdForMathJax(editor, postfix, [["$", "$"], ["\\(","\\)"]]);
);
);
, "mathjax-editing");

StackExchange.ready(function()
var channelOptions =
tags: "".split(" "),
id: "69"
;
initTagRenderer("".split(" "), "".split(" "), channelOptions);

StackExchange.using("externalEditor", function()
// Have to fire editor after snippets, if snippets enabled
if (StackExchange.settings.snippets.snippetsEnabled)
StackExchange.using("snippets", function()
createEditor();
);

else
createEditor();

);

function createEditor()
StackExchange.prepareEditor(
heartbeatType: 'answer',
autoActivateHeartbeat: false,
convertImagesToLinks: true,
noModals: true,
showLowRepImageUploadWarning: true,
reputationToPostImages: 10,
bindNavPrevention: true,
postfix: "",
imageUploader:
brandingHtml: "Powered by u003ca class="icon-imgur-white" href="https://imgur.com/"u003eu003c/au003e",
contentPolicyHtml: "User contributions licensed under u003ca href="https://creativecommons.org/licenses/by-sa/3.0/"u003ecc by-sa 3.0 with attribution requiredu003c/au003e u003ca href="https://stackoverflow.com/legal/content-policy"u003e(content policy)u003c/au003e",
allowUrls: true
,
noCode: true, onDemand: true,
discardSelector: ".discard-answer"
,immediatelyShowMarkdownHelp:true
);



);













draft saved

draft discarded


















StackExchange.ready(
function ()
StackExchange.openid.initPostLogin('.new-post-login', 'https%3a%2f%2fmath.stackexchange.com%2fquestions%2f3145490%2fsum-of-two-continuous-uniform-0-1-random-variables-without-convolution%23new-answer', 'question_page');

);

Post as a guest















Required, but never shown

























2 Answers
2






active

oldest

votes








2 Answers
2






active

oldest

votes









active

oldest

votes






active

oldest

votes









1












$begingroup$

Consider two independent random variables $X_1$ and $X_2$ with uniform distribution over the interval $[0,1]$. Thus we have
$$
f_X_i(x) =
begincases
1 & mboxif x in [0,1]\
0 & mboxotherwise
endcases
$$

for $i=1,2$.
Consider also, e.g., the transformation
begineqnarray
Y_1 &=& X_1+X_2;\
Y_2 &=& X_2;
endeqnarray

Then, as stated by your notes, the joint distribution of $Y_1$ and $Y_2$ is given by
$$
f_Y_1,Y_2(u,v) =
begincases
f_X_1,X_2(u-v,v) =f_X_1(u-v)cdot f_X_2(v) & mboxif (u,v)in [0,1]times[0,1]\
0 & mboxotherwise,
endcases
$$

where we used independence of $X_1$ and $X_2$.



Because you're interested only in the marginal distribution of $Y_1$ we get
$$
f_Y_1(u) = int_0^1 f_X_1(u-v)f_X_2(v) dv,
$$

that is exactly the convolution you were expecting.




EDIT As you requested, I little bit more details. If you use the transformations
begineqnarray
Y_1 &=& g_1(X_1,X_2)\
Y_2 &=& g_2(X_1,X_2)
endeqnarray

you first need to invert them and get
begineqnarray
X_1 &=& h_1(Y_1,Y_2)\
X_2 &=& h_2(Y_1,Y_2).
endeqnarray

Only now, I would recommend, can you



  1. Calculate, from $h_1(u,v)$ and $h_2(u,v)$, $$J = left|beginarraycc fracpartial h_1partial u & fracpartial h_1partial v \ fracpartial h_2partial u & fracpartial h_2partial vendarray right|$$

  2. Determine the joint distribution of $(Y_2,Y_1)$, that, limited to the correct domain, will be given by $$f_Y_1,Y_2(u,v) = |J|cdot f_X_1,X_2(h_1(u,v),h_2(u,v)).$$


In my example
begineqnarray
X_1 &=& Y_1-Y_2\
X_2 &=& Y_2
endeqnarray

are the inverse transformations, so that
begineqnarray
h_1(u,v) &=& u-v\
h_2(u,v) &=& v;
endeqnarray






share|cite|improve this answer











$endgroup$












  • $begingroup$
    Thanks! This makes sense and I think I see what I did wrong. I think I need a variable substitution in limits of my integral to get to the form you have here.
    $endgroup$
    – chris838
    Mar 14 at 12:47






  • 1




    $begingroup$
    @chris838 you're welcome! And: yes, but as you see it would be a little "convoluted" (...) to have a more complicated $Y_2$, since you have to marginalize it "out".
    $endgroup$
    – Matteo
    Mar 14 at 12:49











  • $begingroup$
    Yeh the $Y_2$ makes this much simpler in fact. I'm still stuck on how you get $f_X_1,X_2(u−v, v)$. I end up with $f_X_1,X_2((1/2)(u + v))$ - is that on the right track?
    $endgroup$
    – chris838
    Mar 14 at 13:38






  • 1




    $begingroup$
    @chris838 Ok, I added some more details at the bottom. Are my steps clearer now?
    $endgroup$
    – Matteo
    Mar 14 at 18:54






  • 1




    $begingroup$
    Thanks this is amazing!
    $endgroup$
    – chris838
    Mar 14 at 19:06















1












$begingroup$

Consider two independent random variables $X_1$ and $X_2$ with uniform distribution over the interval $[0,1]$. Thus we have
$$
f_X_i(x) =
begincases
1 & mboxif x in [0,1]\
0 & mboxotherwise
endcases
$$

for $i=1,2$.
Consider also, e.g., the transformation
begineqnarray
Y_1 &=& X_1+X_2;\
Y_2 &=& X_2;
endeqnarray

Then, as stated by your notes, the joint distribution of $Y_1$ and $Y_2$ is given by
$$
f_Y_1,Y_2(u,v) =
begincases
f_X_1,X_2(u-v,v) =f_X_1(u-v)cdot f_X_2(v) & mboxif (u,v)in [0,1]times[0,1]\
0 & mboxotherwise,
endcases
$$

where we used independence of $X_1$ and $X_2$.



Because you're interested only in the marginal distribution of $Y_1$ we get
$$
f_Y_1(u) = int_0^1 f_X_1(u-v)f_X_2(v) dv,
$$

that is exactly the convolution you were expecting.




EDIT As you requested, I little bit more details. If you use the transformations
begineqnarray
Y_1 &=& g_1(X_1,X_2)\
Y_2 &=& g_2(X_1,X_2)
endeqnarray

you first need to invert them and get
begineqnarray
X_1 &=& h_1(Y_1,Y_2)\
X_2 &=& h_2(Y_1,Y_2).
endeqnarray

Only now, I would recommend, can you



  1. Calculate, from $h_1(u,v)$ and $h_2(u,v)$, $$J = left|beginarraycc fracpartial h_1partial u & fracpartial h_1partial v \ fracpartial h_2partial u & fracpartial h_2partial vendarray right|$$

  2. Determine the joint distribution of $(Y_2,Y_1)$, that, limited to the correct domain, will be given by $$f_Y_1,Y_2(u,v) = |J|cdot f_X_1,X_2(h_1(u,v),h_2(u,v)).$$


In my example
begineqnarray
X_1 &=& Y_1-Y_2\
X_2 &=& Y_2
endeqnarray

are the inverse transformations, so that
begineqnarray
h_1(u,v) &=& u-v\
h_2(u,v) &=& v;
endeqnarray






share|cite|improve this answer











$endgroup$












  • $begingroup$
    Thanks! This makes sense and I think I see what I did wrong. I think I need a variable substitution in limits of my integral to get to the form you have here.
    $endgroup$
    – chris838
    Mar 14 at 12:47






  • 1




    $begingroup$
    @chris838 you're welcome! And: yes, but as you see it would be a little "convoluted" (...) to have a more complicated $Y_2$, since you have to marginalize it "out".
    $endgroup$
    – Matteo
    Mar 14 at 12:49











  • $begingroup$
    Yeh the $Y_2$ makes this much simpler in fact. I'm still stuck on how you get $f_X_1,X_2(u−v, v)$. I end up with $f_X_1,X_2((1/2)(u + v))$ - is that on the right track?
    $endgroup$
    – chris838
    Mar 14 at 13:38






  • 1




    $begingroup$
    @chris838 Ok, I added some more details at the bottom. Are my steps clearer now?
    $endgroup$
    – Matteo
    Mar 14 at 18:54






  • 1




    $begingroup$
    Thanks this is amazing!
    $endgroup$
    – chris838
    Mar 14 at 19:06













1












1








1





$begingroup$

Consider two independent random variables $X_1$ and $X_2$ with uniform distribution over the interval $[0,1]$. Thus we have
$$
f_X_i(x) =
begincases
1 & mboxif x in [0,1]\
0 & mboxotherwise
endcases
$$

for $i=1,2$.
Consider also, e.g., the transformation
begineqnarray
Y_1 &=& X_1+X_2;\
Y_2 &=& X_2;
endeqnarray

Then, as stated by your notes, the joint distribution of $Y_1$ and $Y_2$ is given by
$$
f_Y_1,Y_2(u,v) =
begincases
f_X_1,X_2(u-v,v) =f_X_1(u-v)cdot f_X_2(v) & mboxif (u,v)in [0,1]times[0,1]\
0 & mboxotherwise,
endcases
$$

where we used independence of $X_1$ and $X_2$.



Because you're interested only in the marginal distribution of $Y_1$ we get
$$
f_Y_1(u) = int_0^1 f_X_1(u-v)f_X_2(v) dv,
$$

that is exactly the convolution you were expecting.




EDIT As you requested, I little bit more details. If you use the transformations
begineqnarray
Y_1 &=& g_1(X_1,X_2)\
Y_2 &=& g_2(X_1,X_2)
endeqnarray

you first need to invert them and get
begineqnarray
X_1 &=& h_1(Y_1,Y_2)\
X_2 &=& h_2(Y_1,Y_2).
endeqnarray

Only now, I would recommend, can you



  1. Calculate, from $h_1(u,v)$ and $h_2(u,v)$, $$J = left|beginarraycc fracpartial h_1partial u & fracpartial h_1partial v \ fracpartial h_2partial u & fracpartial h_2partial vendarray right|$$

  2. Determine the joint distribution of $(Y_2,Y_1)$, that, limited to the correct domain, will be given by $$f_Y_1,Y_2(u,v) = |J|cdot f_X_1,X_2(h_1(u,v),h_2(u,v)).$$


In my example
begineqnarray
X_1 &=& Y_1-Y_2\
X_2 &=& Y_2
endeqnarray

are the inverse transformations, so that
begineqnarray
h_1(u,v) &=& u-v\
h_2(u,v) &=& v;
endeqnarray






share|cite|improve this answer











$endgroup$



Consider two independent random variables $X_1$ and $X_2$ with uniform distribution over the interval $[0,1]$. Thus we have
$$
f_X_i(x) =
begincases
1 & mboxif x in [0,1]\
0 & mboxotherwise
endcases
$$

for $i=1,2$.
Consider also, e.g., the transformation
begineqnarray
Y_1 &=& X_1+X_2;\
Y_2 &=& X_2;
endeqnarray

Then, as stated by your notes, the joint distribution of $Y_1$ and $Y_2$ is given by
$$
f_Y_1,Y_2(u,v) =
begincases
f_X_1,X_2(u-v,v) =f_X_1(u-v)cdot f_X_2(v) & mboxif (u,v)in [0,1]times[0,1]\
0 & mboxotherwise,
endcases
$$

where we used independence of $X_1$ and $X_2$.



Because you're interested only in the marginal distribution of $Y_1$ we get
$$
f_Y_1(u) = int_0^1 f_X_1(u-v)f_X_2(v) dv,
$$

that is exactly the convolution you were expecting.




EDIT As you requested, I little bit more details. If you use the transformations
begineqnarray
Y_1 &=& g_1(X_1,X_2)\
Y_2 &=& g_2(X_1,X_2)
endeqnarray

you first need to invert them and get
begineqnarray
X_1 &=& h_1(Y_1,Y_2)\
X_2 &=& h_2(Y_1,Y_2).
endeqnarray

Only now, I would recommend, can you



  1. Calculate, from $h_1(u,v)$ and $h_2(u,v)$, $$J = left|beginarraycc fracpartial h_1partial u & fracpartial h_1partial v \ fracpartial h_2partial u & fracpartial h_2partial vendarray right|$$

  2. Determine the joint distribution of $(Y_2,Y_1)$, that, limited to the correct domain, will be given by $$f_Y_1,Y_2(u,v) = |J|cdot f_X_1,X_2(h_1(u,v),h_2(u,v)).$$


In my example
begineqnarray
X_1 &=& Y_1-Y_2\
X_2 &=& Y_2
endeqnarray

are the inverse transformations, so that
begineqnarray
h_1(u,v) &=& u-v\
h_2(u,v) &=& v;
endeqnarray







share|cite|improve this answer














share|cite|improve this answer



share|cite|improve this answer








edited 2 days ago

























answered Mar 13 at 18:32









MatteoMatteo

1,137313




1,137313











  • $begingroup$
    Thanks! This makes sense and I think I see what I did wrong. I think I need a variable substitution in limits of my integral to get to the form you have here.
    $endgroup$
    – chris838
    Mar 14 at 12:47






  • 1




    $begingroup$
    @chris838 you're welcome! And: yes, but as you see it would be a little "convoluted" (...) to have a more complicated $Y_2$, since you have to marginalize it "out".
    $endgroup$
    – Matteo
    Mar 14 at 12:49











  • $begingroup$
    Yeh the $Y_2$ makes this much simpler in fact. I'm still stuck on how you get $f_X_1,X_2(u−v, v)$. I end up with $f_X_1,X_2((1/2)(u + v))$ - is that on the right track?
    $endgroup$
    – chris838
    Mar 14 at 13:38






  • 1




    $begingroup$
    @chris838 Ok, I added some more details at the bottom. Are my steps clearer now?
    $endgroup$
    – Matteo
    Mar 14 at 18:54






  • 1




    $begingroup$
    Thanks this is amazing!
    $endgroup$
    – chris838
    Mar 14 at 19:06
















  • $begingroup$
    Thanks! This makes sense and I think I see what I did wrong. I think I need a variable substitution in limits of my integral to get to the form you have here.
    $endgroup$
    – chris838
    Mar 14 at 12:47






  • 1




    $begingroup$
    @chris838 you're welcome! And: yes, but as you see it would be a little "convoluted" (...) to have a more complicated $Y_2$, since you have to marginalize it "out".
    $endgroup$
    – Matteo
    Mar 14 at 12:49











  • $begingroup$
    Yeh the $Y_2$ makes this much simpler in fact. I'm still stuck on how you get $f_X_1,X_2(u−v, v)$. I end up with $f_X_1,X_2((1/2)(u + v))$ - is that on the right track?
    $endgroup$
    – chris838
    Mar 14 at 13:38






  • 1




    $begingroup$
    @chris838 Ok, I added some more details at the bottom. Are my steps clearer now?
    $endgroup$
    – Matteo
    Mar 14 at 18:54






  • 1




    $begingroup$
    Thanks this is amazing!
    $endgroup$
    – chris838
    Mar 14 at 19:06















$begingroup$
Thanks! This makes sense and I think I see what I did wrong. I think I need a variable substitution in limits of my integral to get to the form you have here.
$endgroup$
– chris838
Mar 14 at 12:47




$begingroup$
Thanks! This makes sense and I think I see what I did wrong. I think I need a variable substitution in limits of my integral to get to the form you have here.
$endgroup$
– chris838
Mar 14 at 12:47




1




1




$begingroup$
@chris838 you're welcome! And: yes, but as you see it would be a little "convoluted" (...) to have a more complicated $Y_2$, since you have to marginalize it "out".
$endgroup$
– Matteo
Mar 14 at 12:49





$begingroup$
@chris838 you're welcome! And: yes, but as you see it would be a little "convoluted" (...) to have a more complicated $Y_2$, since you have to marginalize it "out".
$endgroup$
– Matteo
Mar 14 at 12:49













$begingroup$
Yeh the $Y_2$ makes this much simpler in fact. I'm still stuck on how you get $f_X_1,X_2(u−v, v)$. I end up with $f_X_1,X_2((1/2)(u + v))$ - is that on the right track?
$endgroup$
– chris838
Mar 14 at 13:38




$begingroup$
Yeh the $Y_2$ makes this much simpler in fact. I'm still stuck on how you get $f_X_1,X_2(u−v, v)$. I end up with $f_X_1,X_2((1/2)(u + v))$ - is that on the right track?
$endgroup$
– chris838
Mar 14 at 13:38




1




1




$begingroup$
@chris838 Ok, I added some more details at the bottom. Are my steps clearer now?
$endgroup$
– Matteo
Mar 14 at 18:54




$begingroup$
@chris838 Ok, I added some more details at the bottom. Are my steps clearer now?
$endgroup$
– Matteo
Mar 14 at 18:54




1




1




$begingroup$
Thanks this is amazing!
$endgroup$
– chris838
Mar 14 at 19:06




$begingroup$
Thanks this is amazing!
$endgroup$
– chris838
Mar 14 at 19:06











0












$begingroup$

Yes, you can of course use that approach to get the joint density function, but I don't think your way would be easier to calculate in general.



Regarding your question on $g(y1, y2)$ not being a valid density function. You actually made a mistake here. $g(y_1, y_2)/f(x_1, x_2) = 1/|J|$ rather than $J$.



Just take an example of a single-variable density function ($C$ is the cumulative distribution function)
$$
p(x) = fracdCdx$
$$

$$
p(y) = fracdCdy = fracdCdx cdot fracdxdy = fracp(x)y'(x)
$$






share|cite|improve this answer









$endgroup$












  • $begingroup$
    Hmm, but the notes I refer to explicitly state $g(y_1,y_2) = |J|$ ??
    $endgroup$
    – chris838
    Mar 13 at 12:38











  • $begingroup$
    Possibly you're defining $J$ as $∂(y_1,y_2)/∂(x_1,x_2)$ whereas here I'm defining it as $∂(x_1,x_2)/∂(y_1,y_2)$
    $endgroup$
    – chris838
    Mar 13 at 12:48










  • $begingroup$
    Yes, you are right about the difference in our definition. However, in that case your calculation is wrong $partial(y_1,y_2)/partial(x_1, x_2)=2$ and $partial(x_1, x_2)/partial(y_1, y_2) = 1/2$.
    $endgroup$
    – MoonKnight
    Mar 13 at 16:44










  • $begingroup$
    Apologies - I was playing around with different transform functions. You're right and I've updated the question.
    $endgroup$
    – chris838
    Mar 14 at 12:40






  • 1




    $begingroup$
    your new integral area in $(Y_1, Y_2)$ space is a diamond with 4 cornors (0,0), (1,1), (-1,1), (2, 0), which area is $2$. so it does sum up to $1$.
    $endgroup$
    – MoonKnight
    Mar 14 at 16:47















0












$begingroup$

Yes, you can of course use that approach to get the joint density function, but I don't think your way would be easier to calculate in general.



Regarding your question on $g(y1, y2)$ not being a valid density function. You actually made a mistake here. $g(y_1, y_2)/f(x_1, x_2) = 1/|J|$ rather than $J$.



Just take an example of a single-variable density function ($C$ is the cumulative distribution function)
$$
p(x) = fracdCdx$
$$

$$
p(y) = fracdCdy = fracdCdx cdot fracdxdy = fracp(x)y'(x)
$$






share|cite|improve this answer









$endgroup$












  • $begingroup$
    Hmm, but the notes I refer to explicitly state $g(y_1,y_2) = |J|$ ??
    $endgroup$
    – chris838
    Mar 13 at 12:38











  • $begingroup$
    Possibly you're defining $J$ as $∂(y_1,y_2)/∂(x_1,x_2)$ whereas here I'm defining it as $∂(x_1,x_2)/∂(y_1,y_2)$
    $endgroup$
    – chris838
    Mar 13 at 12:48










  • $begingroup$
    Yes, you are right about the difference in our definition. However, in that case your calculation is wrong $partial(y_1,y_2)/partial(x_1, x_2)=2$ and $partial(x_1, x_2)/partial(y_1, y_2) = 1/2$.
    $endgroup$
    – MoonKnight
    Mar 13 at 16:44










  • $begingroup$
    Apologies - I was playing around with different transform functions. You're right and I've updated the question.
    $endgroup$
    – chris838
    Mar 14 at 12:40






  • 1




    $begingroup$
    your new integral area in $(Y_1, Y_2)$ space is a diamond with 4 cornors (0,0), (1,1), (-1,1), (2, 0), which area is $2$. so it does sum up to $1$.
    $endgroup$
    – MoonKnight
    Mar 14 at 16:47













0












0








0





$begingroup$

Yes, you can of course use that approach to get the joint density function, but I don't think your way would be easier to calculate in general.



Regarding your question on $g(y1, y2)$ not being a valid density function. You actually made a mistake here. $g(y_1, y_2)/f(x_1, x_2) = 1/|J|$ rather than $J$.



Just take an example of a single-variable density function ($C$ is the cumulative distribution function)
$$
p(x) = fracdCdx$
$$

$$
p(y) = fracdCdy = fracdCdx cdot fracdxdy = fracp(x)y'(x)
$$






share|cite|improve this answer









$endgroup$



Yes, you can of course use that approach to get the joint density function, but I don't think your way would be easier to calculate in general.



Regarding your question on $g(y1, y2)$ not being a valid density function. You actually made a mistake here. $g(y_1, y_2)/f(x_1, x_2) = 1/|J|$ rather than $J$.



Just take an example of a single-variable density function ($C$ is the cumulative distribution function)
$$
p(x) = fracdCdx$
$$

$$
p(y) = fracdCdy = fracdCdx cdot fracdxdy = fracp(x)y'(x)
$$







share|cite|improve this answer












share|cite|improve this answer



share|cite|improve this answer










answered Mar 12 at 18:58









MoonKnightMoonKnight

1,646611




1,646611











  • $begingroup$
    Hmm, but the notes I refer to explicitly state $g(y_1,y_2) = |J|$ ??
    $endgroup$
    – chris838
    Mar 13 at 12:38











  • $begingroup$
    Possibly you're defining $J$ as $∂(y_1,y_2)/∂(x_1,x_2)$ whereas here I'm defining it as $∂(x_1,x_2)/∂(y_1,y_2)$
    $endgroup$
    – chris838
    Mar 13 at 12:48










  • $begingroup$
    Yes, you are right about the difference in our definition. However, in that case your calculation is wrong $partial(y_1,y_2)/partial(x_1, x_2)=2$ and $partial(x_1, x_2)/partial(y_1, y_2) = 1/2$.
    $endgroup$
    – MoonKnight
    Mar 13 at 16:44










  • $begingroup$
    Apologies - I was playing around with different transform functions. You're right and I've updated the question.
    $endgroup$
    – chris838
    Mar 14 at 12:40






  • 1




    $begingroup$
    your new integral area in $(Y_1, Y_2)$ space is a diamond with 4 cornors (0,0), (1,1), (-1,1), (2, 0), which area is $2$. so it does sum up to $1$.
    $endgroup$
    – MoonKnight
    Mar 14 at 16:47
















  • $begingroup$
    Hmm, but the notes I refer to explicitly state $g(y_1,y_2) = |J|$ ??
    $endgroup$
    – chris838
    Mar 13 at 12:38











  • $begingroup$
    Possibly you're defining $J$ as $∂(y_1,y_2)/∂(x_1,x_2)$ whereas here I'm defining it as $∂(x_1,x_2)/∂(y_1,y_2)$
    $endgroup$
    – chris838
    Mar 13 at 12:48










  • $begingroup$
    Yes, you are right about the difference in our definition. However, in that case your calculation is wrong $partial(y_1,y_2)/partial(x_1, x_2)=2$ and $partial(x_1, x_2)/partial(y_1, y_2) = 1/2$.
    $endgroup$
    – MoonKnight
    Mar 13 at 16:44










  • $begingroup$
    Apologies - I was playing around with different transform functions. You're right and I've updated the question.
    $endgroup$
    – chris838
    Mar 14 at 12:40






  • 1




    $begingroup$
    your new integral area in $(Y_1, Y_2)$ space is a diamond with 4 cornors (0,0), (1,1), (-1,1), (2, 0), which area is $2$. so it does sum up to $1$.
    $endgroup$
    – MoonKnight
    Mar 14 at 16:47















$begingroup$
Hmm, but the notes I refer to explicitly state $g(y_1,y_2) = |J|$ ??
$endgroup$
– chris838
Mar 13 at 12:38





$begingroup$
Hmm, but the notes I refer to explicitly state $g(y_1,y_2) = |J|$ ??
$endgroup$
– chris838
Mar 13 at 12:38













$begingroup$
Possibly you're defining $J$ as $∂(y_1,y_2)/∂(x_1,x_2)$ whereas here I'm defining it as $∂(x_1,x_2)/∂(y_1,y_2)$
$endgroup$
– chris838
Mar 13 at 12:48




$begingroup$
Possibly you're defining $J$ as $∂(y_1,y_2)/∂(x_1,x_2)$ whereas here I'm defining it as $∂(x_1,x_2)/∂(y_1,y_2)$
$endgroup$
– chris838
Mar 13 at 12:48












$begingroup$
Yes, you are right about the difference in our definition. However, in that case your calculation is wrong $partial(y_1,y_2)/partial(x_1, x_2)=2$ and $partial(x_1, x_2)/partial(y_1, y_2) = 1/2$.
$endgroup$
– MoonKnight
Mar 13 at 16:44




$begingroup$
Yes, you are right about the difference in our definition. However, in that case your calculation is wrong $partial(y_1,y_2)/partial(x_1, x_2)=2$ and $partial(x_1, x_2)/partial(y_1, y_2) = 1/2$.
$endgroup$
– MoonKnight
Mar 13 at 16:44












$begingroup$
Apologies - I was playing around with different transform functions. You're right and I've updated the question.
$endgroup$
– chris838
Mar 14 at 12:40




$begingroup$
Apologies - I was playing around with different transform functions. You're right and I've updated the question.
$endgroup$
– chris838
Mar 14 at 12:40




1




1




$begingroup$
your new integral area in $(Y_1, Y_2)$ space is a diamond with 4 cornors (0,0), (1,1), (-1,1), (2, 0), which area is $2$. so it does sum up to $1$.
$endgroup$
– MoonKnight
Mar 14 at 16:47




$begingroup$
your new integral area in $(Y_1, Y_2)$ space is a diamond with 4 cornors (0,0), (1,1), (-1,1), (2, 0), which area is $2$. so it does sum up to $1$.
$endgroup$
– MoonKnight
Mar 14 at 16:47

















draft saved

draft discarded
















































Thanks for contributing an answer to Mathematics Stack Exchange!


  • Please be sure to answer the question. Provide details and share your research!

But avoid


  • Asking for help, clarification, or responding to other answers.

  • Making statements based on opinion; back them up with references or personal experience.

Use MathJax to format equations. MathJax reference.


To learn more, see our tips on writing great answers.




draft saved


draft discarded














StackExchange.ready(
function ()
StackExchange.openid.initPostLogin('.new-post-login', 'https%3a%2f%2fmath.stackexchange.com%2fquestions%2f3145490%2fsum-of-two-continuous-uniform-0-1-random-variables-without-convolution%23new-answer', 'question_page');

);

Post as a guest















Required, but never shown





















































Required, but never shown














Required, but never shown












Required, but never shown







Required, but never shown

































Required, but never shown














Required, but never shown












Required, but never shown







Required, but never shown







Popular posts from this blog

Lowndes Grove History Architecture References Navigation menu32°48′6″N 79°57′58″W / 32.80167°N 79.96611°W / 32.80167; -79.9661132°48′6″N 79°57′58″W / 32.80167°N 79.96611°W / 32.80167; -79.9661178002500"National Register Information System"Historic houses of South Carolina"Lowndes Grove""+32° 48' 6.00", −79° 57' 58.00""Lowndes Grove, Charleston County (260 St. Margaret St., Charleston)""Lowndes Grove"The Charleston ExpositionIt Happened in South Carolina"Lowndes Grove (House), Saint Margaret Street & Sixth Avenue, Charleston, Charleston County, SC(Photographs)"Plantations of the Carolina Low Countrye

random experiment with two different functions on unit interval Announcing the arrival of Valued Associate #679: Cesar Manara Planned maintenance scheduled April 23, 2019 at 00:00UTC (8:00pm US/Eastern)Random variable and probability space notionsRandom Walk with EdgesFinding functions where the increase over a random interval is Poisson distributedNumber of days until dayCan an observed event in fact be of zero probability?Unit random processmodels of coins and uniform distributionHow to get the number of successes given $n$ trials , probability $P$ and a random variable $X$Absorbing Markov chain in a computer. Is “almost every” turned into always convergence in computer executions?Stopped random walk is not uniformly integrable

How should I support this large drywall patch? Planned maintenance scheduled April 23, 2019 at 00:00UTC (8:00pm US/Eastern) Announcing the arrival of Valued Associate #679: Cesar Manara Unicorn Meta Zoo #1: Why another podcast?How do I cover large gaps in drywall?How do I keep drywall around a patch from crumbling?Can I glue a second layer of drywall?How to patch long strip on drywall?Large drywall patch: how to avoid bulging seams?Drywall Mesh Patch vs. Bulge? To remove or not to remove?How to fix this drywall job?Prep drywall before backsplashWhat's the best way to fix this horrible drywall patch job?Drywall patching using 3M Patch Plus Primer